Recent content by seboastien

  1. S

    Linearising compound pendulum equation

    That's because I thought I was allowed to measure the pendulums mass. Don't worry I've worked it out...finally, turns out I've been overcomplicating things. I'll just plot a graph of h^2 against h*T^2 the y intercept will be -k^2 and the gradient will be g/4pi^2. Thanks anyway.
  2. S

    Linearising compound pendulum equation

    hmmm, my only issue is that its the sqrt of K^2 + h^2 divided by gh it also turns out that k is the radius of gyration and I have no scales to measure the pendulum's mass. I believe I need a y=mx + c where the y intercept will be determined by k, g by m, x by T and h by y. is there any way...
  3. S

    Linearising compound pendulum equation

    I would have to make the axis √((h^2 + K^2)/gh ) but that is a good point. However, I would still like to know how I could linearise it further. I know that a taylor approximation is needed but I don't know how to, or what a value to choose
  4. S

    Linearising compound pendulum equation

    Homework Statement Linearise T=2pi√(K^2 + h^2)/gh K is known constant This is a compound pendulum equation, I want to plot some kind of formula with variable T against some kind of formula with variable H in order to find g from the gradient. Homework Equations The Attempt at a...
  5. S

    What is the Angular Velocity at a Given Angle for a Given Angular Acceleration?

    Yes, that was my problem all along... I forgot to add the constant! Thanks.
  6. S

    What is the Angular Velocity at a Given Angle for a Given Angular Acceleration?

    But then I end up with t=1/2w, how do I find the rest of the answer?
  7. S

    What is the Angular Velocity at a Given Angle for a Given Angular Acceleration?

    Homework Statement angular acceleration of line a= -2w^2 rad/s when theta=pi/6, w= 10 rad/s what is angular velocity when theta= pi/3? Homework Equations chain rule The Attempt at a Solution w= e^-2theta is my answer, but I know this is wrong I know that this question is...
  8. S

    Series expansion of logarithmic function ln(cosx)

    thanks, I'm not entirely certain, but is the differential -tanx?
  9. S

    Series expansion of logarithmic function

    How do I find the first three non-zero terms of ln(5+p), I'm pretty sure that my answer is wrong.
  10. S

    Series expansion of logarithmic function ln(cosx)

    okay, so It's wrong... where exactly have I messed up then?
  11. S

    Power series in real world situation

    Homework Statement A heavy weight is suspended by a cable and pulled to one side by a force F. How much force is required to hold the weight in equilibrium at a given distance x to one side. Tcosθ=W and Tsinθ=F. Find F/W as a power series of θ. Often in a problem like this, what we know is not...
  12. S

    Series expansion of logarithmic function

    Come on guys! I really need to know how to do this!
Back
Top